Những câu hỏi liên quan
Nguyễn Thị Tỉnh
Xem chi tiết
Pham Trong Bach
Xem chi tiết
Cao Minh Tâm
13 tháng 9 2017 lúc 2:43

Chọn B

Bình luận (0)
Pham Trong Bach
Xem chi tiết
Cao Minh Tâm
9 tháng 11 2018 lúc 8:06

Đáp án A.

Bình luận (0)
Pham Trong Bach
Xem chi tiết
Cao Minh Tâm
6 tháng 3 2018 lúc 13:48

Bình luận (0)
Buddy
Xem chi tiết
Hà Quang Minh
24 tháng 8 2023 lúc 0:46

\(a,A=log_23\cdot log_34\cdot log_45\cdot log_56\cdot log_67\cdot log_78\\ =log_28\\ =log_22^3\\ =3\\ b,B=log_22\cdot log_24...log_22^n\\ =log_22\cdot log_22^2...log_22^n\\ =1\cdot2\cdot...\cdot n\\ =n!\)

Bình luận (0)
Pham Trong Bach
Xem chi tiết
Cao Minh Tâm
20 tháng 8 2017 lúc 3:16

Đáp án D.

Ta có

log   6125 7 = log   6125 + log 7 = log 7 2 . 125 + 1 2 log   7

= 5 2 log   7 + log   5 3 = 5 2 n + 3 log   5 = 5 2 n + 3 1 - log   2

= 5 2 n + 3 - 3 m .

Bình luận (0)
Nguyễn Vi
Xem chi tiết
Nghi Minh
22 tháng 6 2019 lúc 15:59

Bạn thử tải app này xem có đáp án không nhé <3 https://giaingay.com.vn/downapp.html

Bình luận (0)
Như Quỳnh
Xem chi tiết
Akai Haruma
12 tháng 11 2018 lúc 19:58

Bài 1:

\(A=\log_380=\log_3(2^4.5)=\log_3(2^4)+\log_3(5)\)

\(=4\log_32+\log_35=4a+b\)

\(B=\log_3(37,5)=\log_3(2^{-1}.75)=\log_3(2^{-1}.3.5^2)\)

\(=\log_3(2^{-1})+\log_33+\log_3(5^2)=-\log_32+1+2\log_35\)

\(=-a+1+2b\)

Bình luận (0)
Akai Haruma
12 tháng 11 2018 lúc 20:05

Bài 2:

\(\log_{30}8=\frac{\log 8}{\log 30}=\frac{\log (2^3)}{\log (10.3)}=\frac{3\log2}{\log 10+\log 3}\)

\(=\frac{3\log (\frac{10}{5})}{1+\log 3}=\frac{3(\log 10-\log 5)}{1+\log 3}=\frac{3(1-b)}{1+a}\)

Bình luận (0)
Akai Haruma
13 tháng 11 2018 lúc 8:35

Bài 3:

\(\log_{27}5=a; \log_87=b; \log_23=c\)

\(\Leftrightarrow \frac{\ln 5}{\ln 27}=a; \frac{\ln 7}{\ln 8}=b; \frac{\ln 3}{\ln 2}=c\)

\(\Leftrightarrow \frac{\ln 5}{\ln (3^3)}=a; \frac{\ln 7}{\ln (2^3)}=b; \ln 3=c\ln 2\)

\(\Leftrightarrow \frac{\ln 5}{3\ln 3}=a; \frac{\ln 7}{3\ln 2}=b; \ln 3=c\ln 2\)

\(\Rightarrow \frac{\ln 5}{3c\ln 2}=a; \frac{\ln 7}{3\ln 2}=b\)

\(\Rightarrow \ln 35=\ln 5+\ln 7=3ac\ln 2+3b\ln 2\)

Do đó:
\(D=\log_6 35=\frac{\ln 35}{\ln 6}=\frac{\ln 35}{\ln 2+\ln 3}=\frac{\ln 35}{\ln 2+c\ln 2}=\frac{3ac\ln 2+3b\ln 2}{\ln 2+c\ln 2}\)

\(=\frac{3ac+3b}{1+c}\)

Bình luận (0)
Buddy
Xem chi tiết
HT.Phong (9A5)
18 tháng 8 2023 lúc 18:23

a) \(log_29\cdot log_34=4\)

b) \(log_{25}\cdot\dfrac{1}{\sqrt{5}}=-\dfrac{1}{4}\)

c) \(log_23\cdot log_9\sqrt{5}\cdot log_54=\dfrac{1}{2}\)

Bình luận (0)
Phạm Đức Huy
Xem chi tiết